LSAT and Law School Admissions Forum

Get expert LSAT preparation and law school admissions advice from PowerScore Test Preparation.

 Administrator
PowerScore Staff
  • PowerScore Staff
  • Posts: 8929
  • Joined: Feb 02, 2011
|
#35021
Complete Question Explanation

Strengthen. The correct answer choice is (B)

Your task in this Strengthen question is to select the answer choice that does most to support the
conclusion that a diet may not have to be extremely low in fat in order to protect the heart.

..... Premise: ..... recent study of 400 North American men and women at risk of second heart
..... ..... ..... ..... attack due to previous heart attack

..... Premise: ..... half put on Mediterranean diet: rich in fish, vegetables, olive oil and grains

..... Premise: ..... half advised to eat more traditional Western diet but to limit fat intake

..... Premise: ..... those following Mediterranean diet were significantly less likely than those on
..... ..... ..... ..... Western to have a second heart attack

..... Premise: ..... but, Mediterranean diet includes a fair amount of fat from fish and olive oil

..... Conclusion: ..... thus, a diet may not have to be extremely low in fat in order to protect the
..... ..... ..... ..... ..... heart

The correct answer in this Strengthen question will support the conclusion that a diet may not have
to be low in fat to protect the heart. The stimulus implies this conclusion may be counter-intuitive,
because the people following a Western diet were told to limit fat intake, implied to be a risk factor
for heart disease. Also, the premise telling us about the fat intake involved in the Mediterranean diet
began with the counter-indicator “but,” following the good news that those on a Mediterranean diet
were less likely to have a second heart attack.

Your prephrase is that a potential weakness in this argument is that while fat intake should be kept
low to protect the heart, the Mediterranean diet “includes a fair amount of fat.” To support the
conclusion, the correct answer could provide a reason why the fat content of the Mediterranean diet
does not reduce the diet’s beneficial effect.

The incorrect answer choices will not support the conclusion, but rather will undermine or have no
effect on it.

Answer choice (A): This choice undermines the conclusion because it emphasizes the benefit of a
very low fat diet.

Answer choice (B): This is the correct answer choice.This choice supports the conclusion by
explaining that the fat included in the Mediterranean diet contains oils that protect the heart against
heart attacks. So, the fat contained in the Mediterranean diet is different than other kinds of fats, and
therefore does not need to be avoided in the same way.

Answer choice (C): This choice has no effect on the conclusion, because it is irrelevant to the
conclusion whether the study participants enjoyed the food or continued eating it after the study was
complete.

Answer choice (D): Since the conclusion concerned only the permissible level of fat in the diet, this
information concerning the addition of exercise to the diet is irrelevant.

Answer choice (E): As with choice (D), this information is irrelevant to the conclusion, which dealt
only with the question of whether a diet has to be extremely low in fact to protect the heart. Whether
the protection offered by the diet might be enhanced by lower blood-cholesterol drugs may be
relevant to a related argument, but not the one in the stimulus.
User avatar
 JNSIWL24
  • Posts: 16
  • Joined: May 10, 2023
|
#107262
Hi,
Is there a GAP located between the last premise and conclusion, in regards to the author’s argument? Are all strengthen questions flawed? If so, what is the flaw committed in this particular question?

Thanks
User avatar
 Jeff Wren
PowerScore Staff
  • PowerScore Staff
  • Posts: 464
  • Joined: Oct 19, 2022
|
#107300
Hi JNSIWL,

The "gap" in the argument is from the results of this particular study to the more general claim made in the conclusion that "a diet may not have to be extremely low in fat in order to protect the heart." In other words, the argument assumes that the results of this study can be generalized to a larger population. Of course, whether or not this is true largely depends on the quality of the study. For example, if the study were somehow flawed, then the results may be completely misleading or incorrect.

Strengthen arguments are not 100% "valid." If they were, there would be no need to strengthen them. You can't really strengthen beyond 100%. However, they aren't necessarily "flawed" in the sense of having a clearly identifiable, classifiable flaw (like what normally occurs in flaw questions). It's probably better to think of strengthen arguments as incomplete. They have partial support for the conclusion, but not 100% support.

For example, I wouldn't describe this argument as outright "flawed," as there is some support for the conclusion and no obvious errors in reasoning (such as any indication that the study itself was flawed). Perhaps 400 people may be too small a sample size, but that isn't really clear and you won't be expected to know exactly what sample size is considered statistically significant. If the flaw is ever too small a sample size, it will be obvious, such as just a few people.
User avatar
 JNSIWL24
  • Posts: 16
  • Joined: May 10, 2023
|
#107317
Thanks so much Jeff! Your response helps and I see your point.

Get the most out of your LSAT Prep Plus subscription.

Analyze and track your performance with our Testing and Analytics Package.